Question and Answers Forum

All Questions      Topic List

None Questions

Previous in All Question      Next in All Question      

Previous in None      Next in None      

Question Number 124192 by Algoritm last updated on 01/Dec/20

Commented by MJS_new last updated on 01/Dec/20

2

$$\mathrm{2} \\ $$

Commented by Algoritm last updated on 01/Dec/20

proof

$$\mathrm{proof} \\ $$

Answered by MJS_new last updated on 01/Dec/20

2<(√3)^(√3) <3  2<3^((√3)/2) <3  4<3^(√3) <9  4^(√3) <27<9^(√3)   (3/2)<(√3)<2 ⇒ {_(9^(3/2) <9^(√3) <9^2  ⇔ 27<9^(√3) <81 ⇒ 27<9^(√3) ) ^(4^(3/2) <4^(√3) <4^2  ⇔ 8<4^(√3) <16 ⇒ 4^(√3) <27)   ⇒ 2<(√3)^(√3) <3 ⇒ ⌊(√3)^(√3) ⌋=2

$$\mathrm{2}<\sqrt{\mathrm{3}}\:^{\sqrt{\mathrm{3}}} <\mathrm{3} \\ $$$$\mathrm{2}<\mathrm{3}^{\frac{\sqrt{\mathrm{3}}}{\mathrm{2}}} <\mathrm{3} \\ $$$$\mathrm{4}<\mathrm{3}^{\sqrt{\mathrm{3}}} <\mathrm{9} \\ $$$$\mathrm{4}^{\sqrt{\mathrm{3}}} <\mathrm{27}<\mathrm{9}^{\sqrt{\mathrm{3}}} \\ $$$$\frac{\mathrm{3}}{\mathrm{2}}<\sqrt{\mathrm{3}}<\mathrm{2}\:\Rightarrow\:\left\{_{\mathrm{9}^{\frac{\mathrm{3}}{\mathrm{2}}} <\mathrm{9}^{\sqrt{\mathrm{3}}} <\mathrm{9}^{\mathrm{2}} \:\Leftrightarrow\:\mathrm{27}<\mathrm{9}^{\sqrt{\mathrm{3}}} <\mathrm{81}\:\Rightarrow\:\mathrm{27}<\mathrm{9}^{\sqrt{\mathrm{3}}} } ^{\mathrm{4}^{\frac{\mathrm{3}}{\mathrm{2}}} <\mathrm{4}^{\sqrt{\mathrm{3}}} <\mathrm{4}^{\mathrm{2}} \:\Leftrightarrow\:\mathrm{8}<\mathrm{4}^{\sqrt{\mathrm{3}}} <\mathrm{16}\:\Rightarrow\:\mathrm{4}^{\sqrt{\mathrm{3}}} <\mathrm{27}} \right. \\ $$$$\Rightarrow\:\mathrm{2}<\sqrt{\mathrm{3}}\:^{\sqrt{\mathrm{3}}} <\mathrm{3}\:\Rightarrow\:\lfloor\sqrt{\mathrm{3}}\:^{\sqrt{\mathrm{3}}} \rfloor=\mathrm{2} \\ $$

Commented by MJS_new last updated on 01/Dec/20

same procedure  6<3^(√3) <7  6^(√3) <27<7^(√3)   ((17)/(10))<(√3)<((18)/(10)) ⇒ {_(7^((17)/(10)) <7^(√3) <7^((18)/(10)) ) ^(6^((17)/(10)) <6^(√3) <6^((18)/(10)) )   6^((18)/(10)) <27 ⇔ 6^(18) <27^(10)  ⇔ 2^(18) 3^(18) <3^(12) 3^(18)  ⇔  ⇔ 2^(18) <3^(12)  ⇔ 2(√2)<3 true  27<7^((17)/(10))  ⇔ 27^(10) <7^(17)  ⇔ 205891132094649<232630513987207  true (sorry I found no better way)  ⇒ 6^(√3) <6^((18)/(10)) <27<7^((17)/(10)) <7^(√3)   ⇔ 6<3^(√3) <7

$$\mathrm{same}\:\mathrm{procedure} \\ $$$$\mathrm{6}<\mathrm{3}^{\sqrt{\mathrm{3}}} <\mathrm{7} \\ $$$$\mathrm{6}^{\sqrt{\mathrm{3}}} <\mathrm{27}<\mathrm{7}^{\sqrt{\mathrm{3}}} \\ $$$$\frac{\mathrm{17}}{\mathrm{10}}<\sqrt{\mathrm{3}}<\frac{\mathrm{18}}{\mathrm{10}}\:\Rightarrow\:\left\{_{\mathrm{7}^{\frac{\mathrm{17}}{\mathrm{10}}} <\mathrm{7}^{\sqrt{\mathrm{3}}} <\mathrm{7}^{\frac{\mathrm{18}}{\mathrm{10}}} } ^{\mathrm{6}^{\frac{\mathrm{17}}{\mathrm{10}}} <\mathrm{6}^{\sqrt{\mathrm{3}}} <\mathrm{6}^{\frac{\mathrm{18}}{\mathrm{10}}} } \right. \\ $$$$\mathrm{6}^{\frac{\mathrm{18}}{\mathrm{10}}} <\mathrm{27}\:\Leftrightarrow\:\mathrm{6}^{\mathrm{18}} <\mathrm{27}^{\mathrm{10}} \:\Leftrightarrow\:\mathrm{2}^{\mathrm{18}} \mathrm{3}^{\mathrm{18}} <\mathrm{3}^{\mathrm{12}} \mathrm{3}^{\mathrm{18}} \:\Leftrightarrow \\ $$$$\Leftrightarrow\:\mathrm{2}^{\mathrm{18}} <\mathrm{3}^{\mathrm{12}} \:\Leftrightarrow\:\mathrm{2}\sqrt{\mathrm{2}}<\mathrm{3}\:\mathrm{true} \\ $$$$\mathrm{27}<\mathrm{7}^{\frac{\mathrm{17}}{\mathrm{10}}} \:\Leftrightarrow\:\mathrm{27}^{\mathrm{10}} <\mathrm{7}^{\mathrm{17}} \:\Leftrightarrow\:\mathrm{205891132094649}<\mathrm{232630513987207} \\ $$$$\mathrm{true}\:\left(\mathrm{sorry}\:\mathrm{I}\:\mathrm{found}\:\mathrm{no}\:\mathrm{better}\:\mathrm{way}\right) \\ $$$$\Rightarrow\:\mathrm{6}^{\sqrt{\mathrm{3}}} <\mathrm{6}^{\frac{\mathrm{18}}{\mathrm{10}}} <\mathrm{27}<\mathrm{7}^{\frac{\mathrm{17}}{\mathrm{10}}} <\mathrm{7}^{\sqrt{\mathrm{3}}} \\ $$$$\Leftrightarrow\:\mathrm{6}<\mathrm{3}^{\sqrt{\mathrm{3}}} <\mathrm{7} \\ $$

Commented by Algoritm last updated on 01/Dec/20

Brilliant

$$\mathrm{Brilliant} \\ $$

Commented by Algoritm last updated on 01/Dec/20

Commented by MJS_new last updated on 01/Dec/20

what do you mean?  3^(√3) ≈6.70499

$$\mathrm{what}\:\mathrm{do}\:\mathrm{you}\:\mathrm{mean}? \\ $$$$\mathrm{3}^{\sqrt{\mathrm{3}}} \approx\mathrm{6}.\mathrm{70499} \\ $$

Commented by Algoritm last updated on 01/Dec/20

Terms of Service

Privacy Policy

Contact: info@tinkutara.com